Limit Continuity and Differentiability 7 Question 28

28. Let $f: R \rightarrow R, g: R \rightarrow R$ and $h: R \rightarrow R$ be differentiable functions such that $f(x)=x^{3}+3 x+2, g(f(x))=x$ and $h(g(g(x)))=x$ for all $x \in R$. Then,

(2016 Adv.)

(a) $g^{\prime}(2)=\frac{1}{15}$

(b) $h^{\prime}(1)=666$

(c) $h(0)=16$

(d) $h(g(3))=36$

Show Answer

Answer:

Correct Answer: 28. (b)

Solution:

  1. $y=\frac{a x^{2}}{(x-a)(x-b)(x-c)}+\frac{b x}{(x-b)(x-c)}+\frac{c}{(x-c)}+1$

$$ =\frac{a x^{2}}{(x-a)(x-b)(x-c)}+\frac{b x}{(x-b)(x-c)}+\frac{x}{(x-c)} $$

$$ \begin{aligned} & =\frac{a x^{2}}{(x-a)(x-b)(x-c)}+\frac{x}{(x-c)} \frac{b}{x-b}+1 \\ & =\frac{a x^{2}}{(x-a)(x-b)(x-c)}+\frac{x}{(x-c)} \cdot \frac{x}{(x-b)} \\ & =\frac{x^{2}}{(x-c)(x-b)} \frac{a}{x-1}+1 \Rightarrow y=\frac{x^{3}}{(x-a)(x-b)(x-c)} \end{aligned} $$

$\Rightarrow \quad \log y=\log x^{3}-\log (x-a)(x-b)(x-c)$

$\Rightarrow \log y=3 \log x-\log (x-a)-\log (x-b)-\log (x-c)$

On differentiating, we get

$$ \begin{array}{rlrl} \frac{y^{\prime}}{y} & =\frac{3}{x}-\frac{1}{x-a}-\frac{1}{x-b}-\frac{1}{x-c} \\ \Rightarrow & \frac{y^{\prime}}{y} & =\frac{1}{x}-\frac{1}{x-a}+\frac{1}{x}-\frac{1}{x-b}+\frac{1}{x}-\frac{1}{x-c} \\ \Rightarrow & \frac{y^{\prime}}{y} & =\frac{-a}{x(x-a)}-\frac{b}{x(x-b)}-\frac{c}{x(x-c)} \\ \Rightarrow & \frac{y^{\prime}}{y} & =\frac{a}{x(a-x)}+\frac{b}{x(b-x)}+\frac{c}{x(c-x)} \\ \Rightarrow & \frac{y^{\prime}}{y} & =\frac{1}{x} \frac{a}{a-x}+\frac{b}{b-x}+\frac{c}{c-x} \end{array} $$



जेईई के लिए मॉक टेस्ट

एनसीईआरटी अध्याय वीडियो समाधान

दोहरा फलक